Matrix dimension of a vector in R4

Click For Summary
The discussion revolves around calculating the dimension of the subspace W in R4 defined by the equation V^T X = 0, where V = (1, 2, -3, -1)^T. The initial attempt suggested that the dimension might be 1, but further analysis shows that the equation a + 2b - 3c - d = 0 can be expressed in terms of three free variables (b, c, d). This leads to a solution that indicates the dimension of W is actually 3, as it can be spanned by three independent vectors. The final conclusion confirms that the dimension of the subspace W is 3.
Nicksto
Messages
2
Reaction score
0

Homework Statement


Let W be the subspace of R4 defined by W={x:V^TX=0}. Calculate dim(w) where
V=(1 2 -3 -1)^T
note: V^T means V Transpose, sorry I don't know how to do transpose sign in here.


Homework Equations





The Attempt at a Solution


I tries to do it (1 2 -3 -1)(x1 x2 x3 x4)^T=0
x1+2(x2)-3(x3)-x4=0
So does it means dim(w)=1 ?
 
Physics news on Phys.org
Nicksto said:

Homework Statement


Let W be the subspace of R4 defined by W={x:V^TX=0}. Calculate dim(w) where
V=(1 2 -3 -1)^T
note: V^T means V Transpose, sorry I don't know how to do transpose sign in here.


Homework Equations





The Attempt at a Solution


I tries to do it (1 2 -3 -1)(x1 x2 x3 x4)^T=0
x1+2(x2)-3(x3)-x4=0
So does it means dim(w)=1 ?

No. I will use a,b,c,d instead of the subscripted x's to save typing. You have ##a+2b-3c-d=0##. Solving for ##a## gives ##a=-2b+3c+d##. So$$
\left(\begin{array}{c} a \\ b \\ c \\d \end{array}\right) =
\left(\begin{array}{c} -2b+3c+d \\ b \\ c \\d \end{array}\right) =
b\left(\begin{array}{c} -2 \\ 1 \\ 0 \\0 \end{array}\right) +
c\left(\begin{array}{c} 3 \\ 0 \\ 1 \\0 \end{array}\right) +
d\left(\begin{array}{c} 1 \\ 0 \\ 0 \\1 \end{array}\right)

$$
Does that give you a hint about the dimension?
 
Last edited:
It's 3 right? thanks man
 
Question: A clock's minute hand has length 4 and its hour hand has length 3. What is the distance between the tips at the moment when it is increasing most rapidly?(Putnam Exam Question) Answer: Making assumption that both the hands moves at constant angular velocities, the answer is ## \sqrt{7} .## But don't you think this assumption is somewhat doubtful and wrong?

Similar threads

  • · Replies 5 ·
Replies
5
Views
3K
  • · Replies 23 ·
Replies
23
Views
3K
  • · Replies 9 ·
Replies
9
Views
3K
Replies
15
Views
2K
Replies
10
Views
3K
  • · Replies 15 ·
Replies
15
Views
2K
Replies
8
Views
2K
Replies
3
Views
2K
  • · Replies 24 ·
Replies
24
Views
3K
  • · Replies 2 ·
Replies
2
Views
3K